You are on page 1of 55

INSIGHTS

 MOCK  TEST  SERIES  2015:  TEST  –  30  SOLUTIONS  


 
1. Solution: c)

Jaya Prakash once stated that Total Revolution is a combination of seven revolutions, viz.,
political, social, economic, cultural, ideological or intellectual, educational and spiritual; and the
main motive being to bring in a change in the existing society that is in tune with the ideals of the
Sarvodaya.

JP had a very idealistic notion of society and it is in this endeavor, he shifted from Marxism to
Socialism and later towards Sarvodaya.

By the early 1970s, JP completely withdrew from party and power politics, and concentrated more
on social regeneration through peaceful means.

JP Narayanan has been in news for some time.

2. Solution: a)

http://timesofindia.indiatimes.com/city/varanasi/Jaitleys-Buddha-Circuit-plan-is-a-
masterstroke/articleshow/38184422.cms

The complete map of the Buddhist Circuit comprises Lumbini in Nepal, Bodh Gaya, Vaishali and
Rajgir in Bihar and Sarnath in Varanasi, Shravasti and Kushinagar.

The Buddh International Circuit is also a motor racing circuit in Greater Noida, Uttar Pradesh. The
circuit is best known as the venue for the annual Formula One Indian Grand Prix, which was first
hosted in October 2011 but has been suspended since the 27th of October 2013 due to an ongoing
tax dispute with the Uttar Pradesh government.

3. Solution: c)

This is a centenary (100 years) for Tagore’s Gitanjali Noble prize award. These lines are the very
first lines of Gitanjali. Hence, such unexpected questions may be expected by UPSC.

http://www.thehindu.com/features/friday-review/music/1000-voices-bengali-choir-celebrate-
100-years-of-tagores-gitanjali/article7382975.ece

You can read interesting stuff about Gitanjali here (not very useful for the exam though) -
http://www.sacred-texts.com/hin/tagore/gitnjali.htm

4. Solution: b)

http://www.insightsonindia.com                                                                              INSIGHTS   Page  1  


 
INSIGHTS  MOCK  TEST  SERIES  2015:  TEST  –  30  SOLUTIONS  
 
As per the census there are 24.39 crore total number of households in the country, of which 17.91
crore live in villages. Of these, 10.69 crore households are considered as deprived. 50 percent or
5.37 crore deprived households in rural areas are landless and depend on manual labour for
livelihood.

House hold size: Average size of a rural Indian household is 4.93. Highest in Uttar Pradesh at 6.26.
Lowest in Andhra Pradesh at 3.86.

Male dominance: majority of 87 per cent households is male-headed. Nearly 13 per cent do have a
female head. In rural areas of Rajasthan, around 91 per cent households are headed by men. In
Kerala 26 per cent are women-headed households (highest among the states).

Literacy: Over one-third of population living in rural areas is illiterate. Higher literacy in terms of
percentage is in Kerala (88.62), Delhi (86.42), Goa (84.58), Sikkim (79.88) and Himachal Pradesh
(77.95). Lowest literacy is in Rajasthan (58), Madhya Pradesh (44.19), Bihar (43.85), Telangana
(40.42) and Chattisgarh (39.59). SC and ST Population:Large proportion of the households across
the country belong to the SC and ST category i.e. about 30 per cent of rural households.

Employment: Rural India is largely self-employed or employed in the unorganised sector. Around
10 per cent households are on salaried jobs of which the majority are in government jobs.

5. Solution: c)

Forex reserves do not serve any useful purpose apart from acting as a hedge against BoP crisis; to
maintain the value of rupee in international market in case of excessive volatility; and to undertake
sterilization in the economy.

Therefore, there was a proposal to use them for infrastructure projects.

While the proposal look simple and straight enough, there are a number of strategic considerations
like nature of financing; risk in projects; BoP status; effect on money supply and inflation etc.

A decision should be made considering all the above factors.

http://www.thehindu.com/news/national/centre-wants-to-break-forex-
reserves/article7400569.ece

6. Solution: a)

Eligibility to get rights under the Act is confined to those who "primarily reside in forests" and who
depend on forests and forest land for a livelihood. Further, either the claimant must be a member
of the Scheduled Tribes scheduled in that area or must have been residing in the forest for 75 years

• Title rights - i.e. ownership - to land that is being farmed by tribals or forest dwellers as on
13 December 2005, subject to a maximum of 4 hectares; ownership is only for land that is
actually being cultivated by the concerned family as on that date, meaning that no new
lands are granted
• Use rights - to minor forest produce (also including ownership), to grazing areas, to
pastoralist routes, etc.

http://www.insightsonindia.com                                                                              INSIGHTS   Page  2  


 
INSIGHTS  MOCK  TEST  SERIES  2015:  TEST  –  30  SOLUTIONS  
 
• Relief and development rights - to rehabilitation in case of illegal eviction or forced
displacement; and to basic amenities, subject to restrictions for forest protection
• Forest management rights - to protect forests and wildlife

7. Solution: d)

Almost all States have performed poorly in reducing the number of underweight adolescent girls.

Uttar Pradesh still has the highest levels of child stunting, with over 50 per cent of the children
under the age of five underdeveloped, meaning that their height is more than two standard
deviations less than the expected height for their age for that population.

Jharkhand, meanwhile, has the highest number of underweight children under the age of five,
meaning their weight for age is more than two standard deviations less than what would be
expected.

Kerala remains the best performing State in the number of child stunting cases, while Manipur and
Mizoram have the lowest numbers of underweight children.

8. Solution: d)

The stabilizing tendency of the market is often know as the “invisible hand”.

When there is excess supply or lower demand for a commodity, price of a commodity lowers. This
is turn revives the demand in the market and increases production of the commodity. The reverse
is also true.

Optimizing behaviour of the individuals means that every individual acts rationally in their
maximum self-interest. This is at the core of the principle of market forces.

Perfect competition ensures that all such transactions take place in an unrestricted and non-
monopoly markets.

9. Solution: c)

http://www.insightsonindia.com                                                                              INSIGHTS   Page  3  


 
INSIGHTS  MOCK  TEST  SERIES  2015:  TEST  –  30  SOLUTIONS  
 
http://timesofindia.indiatimes.com/city/chandigarh/Haryanas-Bhirrana-oldest-Harappan-site-
Rakhigarhi-Asias-largest-ASI/articleshow/46926693.cms

Rakhigarhi, or Rakhi Garhi is a village in Hisar District in the state of Haryana in India, situated in
the north-west about 150 kilometers from Delhi. In 1963, archaeologists discovered that this place
was the site of the largest known city of the Indus-Sarasvati civilization, much larger and ancient
than Harappa and Mohenjodaro sites.

It is situated on the dry bed of the Sarasvati river, which is believed to have once flown through
this place and dried up by 2000 BC.

According to the archaeologists, Rakhigarhi is an ideal nucleus from where the Harappan
civilisation began in the Ghaggar basin in Haryana and gradually grew from here and slowly
expanded to the Indus valley.

10. Solution: a)

http://www.thehindu.com/opinion/op-ed/prime-minister-narendra-modis-central-asia-
visit/article7396013.ece

Here is the text from the article which discusses radicalism in Central Asia:

“Prime Minister Narendra Modi’s eight-day visit to the five Central Asian States — Uzbekistan,
Kazakhstan, Kyrgyzstan, Tajikistan and Turkmenistan — is taking place at the most opportune
juncture, for Mr. Modi has indicated that he proposes to focus on the radical Islamist threat to the
region. Given the kind of extremist winds sweeping across the region, the Muslim populations of
these states face uncertain times. States such as Tajikistan are especially vulnerable, as many from
the ranks of their security agencies are beginning to join the Islamic State (IS).

The threats that these states face from radical Islamist elements are, indeed, real. At the same
time, it is also significant that the leadership of these Central Asian States should look to India to
provide them with answers on how to insulate their Muslim populations from these kinds of
threats. India’s success, to date, in insulating its own Muslim population from such radicalism has
gained wide acceptance, even as the so-called ‘counter radicalisation’ programmes followed in the
West are proving to be a failure.”

11. Solution: d)

http://www.forbes.com/sites/timworstall/2015/04/18/indias-change-in-gdp-calculation-method-
seems-highly-sensible/

The CSO is responsible for coordination of statistical activities in the country, including National
Income Accounting; conduct of Annual Survey of Industries, Economic Censuses, compilation of
Index of Industrial Production, Consumer Price Indices etc. The most important indices used for
compilation of GDP are Index of Industrial Production (IIP), Wholesale Price Index (WPI), and
Consumer Price Indices – for Industrial Workers (CPI-IW), Agricultural Labourers (CPI-AL) and
Rural Labourers (CPI-RL).

http://www.insightsonindia.com                                                                              INSIGHTS   Page  4  


 
INSIGHTS  MOCK  TEST  SERIES  2015:  TEST  –  30  SOLUTIONS  
 
This data is compiled by central and state ministries, independent departments, and other
governmental organisations. For example, the Industrial Statistics Unit at the Department of
Industrial Policy & Promotion, Ministry of Commerce & Industry, provides production data to the
CSO for compiling IIP. Similarly, commodity-wise data on import-export, production, crop, area,
WPI, CPI etc. is collected and analysed by the Price Monitoring Cell, Department of Consumer
Affairs, Ministry of Consumer Affairs, Food and Public Distribution.

12. Solution: a)

Article 29 provides that any section of the citizens residing in any part of India having a distinct
language, script or culture of its own, shall have the right to conserve the same.

Further, no citizen shall be denied admission into any educational institution maintained by the
State or receiving aid out of State funds on grounds only of religion, race, caste, or language.

Their rights are to be protected because their future in a democracy is vulnerable to the will of the
majority.

13. Solution: d)

The exact cause of the stock market crash is not clear. It is still being probed. However, speculative
trading is likely to be blamed.

Read here to understand what is the impact of the crash and what can be the possible causes
http://economictimes.indiatimes.com/markets/stocks/news/chinese-police-find-clues-of-3-2-
trillion-stock-market-crash/articleshow/48042250.cms

Stocks finally surged few days ago due to several aggressive countermeasures taken by Chinese
authorities.

These countermeasures include an interest rate cut, curbing IPOs, loosening margin requirements,
allowing the use of property as collateral for margin loans, and encouraging brokerage firms to buy
stocks with cash from the People’s Bank of China. Short sellers have even been threatened with
arrest, which has raised eyebrows around the world.

14. Solution: b)

The powerful Typhoon Linfa has hit the southern China after it made landfall on southern
province of Guangdong.

Typhoon Linfa has hit the southern province by bringing high speed winds along with heavy rain.
However after it had made landfall, the typhoon has lost strength rapidly after it passed near Hong
Kong due to weakened low pressure area.

Typhoon Linfa is the first of two large storms Chan-Hom and Nangka which are going to hit China.
The third storm, Nangka is the most powerful among them and has reached supertyphoon
strength with winds of 250 km/hr. But it has strong presence far out in the Pacific Ocean and not
expected to hit any land with high strength.
http://www.insightsonindia.com                                                                              INSIGHTS   Page  5  
 
INSIGHTS  MOCK  TEST  SERIES  2015:  TEST  –  30  SOLUTIONS  
 
15. Solution: d)

It was indigenously developed by the DRDO and Bharat Electronics Ltd (BEL) as part of
Integrated Guided Missile Development Programme (IGMDP) of the 1980s.

• Manufactured using 92 per cent of indigenous machinery and can be transported anywhere
via road, water or air transport.
• It is a potent supersonic mobile multi-target point, multi-directional missile, capable to
engage multiple air targets simultaneously using sophisticated multi-function phased array
radars.
• Capable to neutralise aerial targets like cruise missiles, fighter jets and air-to-surface
missiles with strike range of 25km.
• It can destroy aerial target upto 30 km away at altitudes upto 18,000 m.
• It has supersonic speed i.e. it is three times faster than that of sound and capable to carry
conventional warhead of 60 kg.

16. Solution: a)

The first system has been installed in its capital Dehradun which will issue warnings 1-40 seconds
before earthquakes of magnitude 5 or more occur.

The system is designed and manufactured by Italian firm, Space Dynamics which already has
installed it in Japan, Italy and US.

The system is incorporated with sensors which are capable to detect P (Primary) and S
(Secondary) waves generated during an earthquake. It is capable to detect the harmless P wave,
which travels faster than the S wave for advance warning. The system has the earliest detection
among those currently in use worldwide.

17. Solution: a)

Under the provisions of FATCA, it is mandatory for all financial institutions outside of the US to
periodically transmit information on financial accounts held by US citizens to its US Internal
Revenue Service.

The agreement will promote transparency on tax matters and curb offshore tax evasion by
exchange of information between the two countries. It is obligatory on signatories to exchange a
wide range of financial information among themselves automatically and periodically.

Henceforth, Indian financial institutions will have to reveal information about US tax payers to the
revenue department. This information will be further passed on to the US tax authorities. In turn,
the US will also share similar kind of financial information with India.

18. Solution: d)

India and Kazakhstan have recently signed five agreements in various sectors focusing on
strategic, economic and trade cooperation between both nations.

http://www.insightsonindia.com                                                                              INSIGHTS   Page  6  


 
INSIGHTS  MOCK  TEST  SERIES  2015:  TEST  –  30  SOLUTIONS  
 
Kazakhstan the world’s largest producer of uranium will supply a total of 5,000 tonnes of the
natural uranium to India during the 2015-19. Thus it will play important role in India’s energy
requirement and energy security.

Apart from signing these agreements, India has also shown interest in fertilizer sector in
Kazakhstan due to abundance availability of Natural Gas; and hydro carbon sectors of Kazakhstan
for drilling and production of crude oil.

19. Solution: d)

These villages will be developed under the Border Area Development Programme (BADP) with
special emphasis on promotion of sports activities and rural tourism.

The funds under BADP for this initiative will be provided to the states as a 100 per cent non-
lapsable special central assistance.

It will be supplemented from the budgetary allocation of Rs. 990 crore for the financial year 2015-
16. The development of villages under BADP will include schemes such as Swachh Bharat Abhiyan,
Promotion of sports activities in border areas.

Skill Development Programmes along with promotion of rural tourism (i.e. border tourism) and
protection of heritage sites.

It will also cover construction of helipads in remote and inaccessible hilly areas, which do not have
road connectivity, skill development training to farmers for use of modern or scientific technique
in farming, organic farming etc.

20. Solution: a)

Japan’s Meiji-era (1868-1912) industrial revolution sites have received World Cultural Heritage
status from United Nations Educational, Scientific and Cultural Organization (UNESCO).

Located southwest of Japan, it includes series of eleven properties that comprises Iron and Steel,
coalmines and shipyards industries. It contributed in Japan’s evolution from feudalism to a
successful industrialised modern economy.

It traces the development phase of Japan’s heavy industries and its integration to Western
technology along with fundamental changes in its traditional culture.

21. Solution: c)

In economics, the Phillips curve is a historical inverse relationship between rates of unemployment
and corresponding rates of inflation that result in an economy. Stated simply, decreased
unemployment, (i.e., increased levels of employment) in an economy will correlate with higher
rates of inflation.

The topic has been asked by UPSC a few times in Prelims as well as Mains, hence its important to
know the concept.
http://www.insightsonindia.com                                                                              INSIGHTS   Page  7  
 
INSIGHTS  MOCK  TEST  SERIES  2015:  TEST  –  30  SOLUTIONS  
 
22. Solution: b)

One of the policy objectives of RBI is to ensure high-quality corporate governance in banks. RBI
has issued guidelines for ‘fit and proper’ criteria for director of banks. One of these guidelines is
that the directors of the banks should have special knowledge / experience in the various banking
related areas. RBI can also appoint additional directors to the board of a banking company, and
supersede the decisions of the boards.

The interest rates on most of the categories of deposits and lending transactions have been
deregulated and are largely determined by banks. Reserve Bank regulates the interest rates on
savings bank accounts and deposits of non-resident Indians (NRI), small loans up to rupees two
lakh, export credits and a few other categories of advances.

23. Solution: b)

The system of gold pegging was abolished long time ago.

There is some confusion regarding the interchange of the currency with gold, but the system that
India follows is that money cannot be exchanged for gold under any circumstances due to gold's
lack of liquidity; therefore, money cannot be changed into gold by the RBI. India follows the same
principle as Great Britain and the US.

Reserve Bank of India clarifies its position regarding the promissory clause printed on each
banknote:

"As per Section 26 of Reserve Bank of India Act, 1934, the Bank is liable to pay the value of
banknote. This is payable on demand by RBI, being the issuer. The Bank's obligation to pay the
value of banknote does not arise out of a contract but out of statutory provisions.The promissory
clause printed on the banknotes i.e., "I promise to pay the bearer an amount of X" is a statement
which means that the banknote is a legal tender for X amount. The obligation on the part of the
Bank is to exchange a banknote for coins of an equivalent amount.”

24. Solution: d)

Apart from the product method, there are two other ways of GDP calculation.

Income Method:

Under this method, national income is measured as a flow of factor incomes. There are generally
four factors of production labour, capital, land and entrepreneurship. Labour gets wages and
salaries, capital gets interest, land gets rent and entrepreneurship gets profit as their
remuneration.

http://www.insightsonindia.com                                                                              INSIGHTS   Page  8  


 
INSIGHTS  MOCK  TEST  SERIES  2015:  TEST  –  30  SOLUTIONS  
 
Besides, there are some self-employed persons who employ their own labour and capital such as
doctors, advocates, CAs, etc. Their income is called mixed income. The sum-total of all these factor
incomes is called NDP at factor costs.

Expenditure Method:

In this method, national income is measured as a flow of expenditure. GDP is sum-total of private
consumption expenditure. Government consumption expenditure, gross capital formation
(Government and private) and net exports (Export-Import).

25. Solution: d)

This article explains the impact of subsidy very well. Go through the entire article.

http://www.business-standard.com/article/opinion/rajiv-shastri-why-india-s-subsidies-are-
inflationary-114020200738_1.html

26. Solution: d)

Greater FDI, FII and other capital flows inside the country become more profitable if the interest
rates are low.

For e.g. a foreign investor will be able to earn greater returns if the investment money is available
at a lower interest rate.

Moreover, NRIs are less likely to keep their deposits in India if the interest rate offered is not
competitive as compared to their present residence country.

27. Solution: d)

In economics, a transfer payment (or government transfer or simply transfer) is a redistribution of


income in the market system. These payments are considered to be non-exhaustive because they
do not directly absorb resources or create output.

Transfer payments are made to individuals through programs such as Social Security, Welfare and
Veteran's benefits.

Pensions and other such schemes that benefit the public are also covered under transfer payments.

28. Solution: d)

Refer to this article for understanding the major issues associated with the scheme.

http://www.firstpost.com/business/economy/gold-monetization-scheme-why-offering-higher-
rate-of-interest-is-critical-for-its-success-2284738.html

Under the GMS, an individual or entity can walk into a test centre and gets the gold melted, purity
assessed and converted into bars, against which the bank will issue a certificate to the holder. On
this deposit, the customer earns an interest decided by the bank, which will be exempted from
http://www.insightsonindia.com                                                                              INSIGHTS   Page  9  
 
INSIGHTS  MOCK  TEST  SERIES  2015:  TEST  –  30  SOLUTIONS  
 
income, wealth or capital gains taxes. On maturity, the customer can get the gold or cash back plus
interest amount.

High inflation will erode the interest rate offered on the gold and people will again flock back to
Gold.

Also public emotional attachment to gold may not allow them to melt the metal and convert it into
gold bars.

Statement 3 is already taken care in the scheme.

29. Solution: d)

All the members of a municipality shall be elected directly by the people of the municipal area. For
this purpose, each municipal area shall be divided into territorial constituencies to be known as
wards.

The State government does not directly appoint the members. The state legislature may only
provide for the manner of reservation of offices of chairpersons in the municipalities for SCs, STs
and women.

The state legislature may (a) authorise a municipality to levy, collect and appropriate taxes, duties,
tolls and fees; (b) assign to a municipality taxes, duties, tolls and fees levied and collected by state
government; (c) provide for making grants-in-aid to the municipalities from the consolidated fund
of the state; and (d) provide for constitution of funds for crediting all moneys of the municipalities.

30. Solution: c)

The Assembly was made a fully sovereign body, which could frame any Constitution it pleased. The
act empowered the Assembly to abrogate or alter any law made by the British Parliament in
relation to India.

The Assembly also became a legislative body. In other words, two separate functions were assigned
to the Assembly, i.e. making of a constitution for free India and enacting of ordinary laws for the
country.

In the CA, the representatives of each community were to be elected by members of that
community in the provincial legislative assembly and voting was to be by the method of
proportional representation by means of single transferable vote.

The representatives of princely states were to be nominated by the heads of the princely states. It is
thus clear that the Constituent Assembly was to be a partly elected and partly nominated body.

The PM was appointed by the party in majority.

http://www.insightsonindia.com                                                                              INSIGHTS   Page  10  


 
INSIGHTS  MOCK  TEST  SERIES  2015:  TEST  –  30  SOLUTIONS  
 
31. Solution: d)

The government may resign only if it fails to pass the budget in the Parliament. Failure in passing
regular public bills is not considered a defeat of the government. Passing of a no-confidence
motion does.

PM can be the member of any house. It is not a convention that he must be from the lower house,
neither has it been codified.

Statement 3 is more of a political tool rather than a codified convention.

32. Solution: b)

The very existence of the council depends on the will of the assembly. The council can be abolished
by the Parliament on the recommendation of the assembly.

The position of the council vis-a-vis the assembly is much weaker than the position of the Rajya
Sabha vis-a-vis the Lok Sabha.

The council is subordinate to the assembly in all respects.

33. Solution: d)

This issue was in news because of government planning to introduce Yoga in school curriculums;
and some Indian cricket fans being booked for sedition for supporting Pakistani cricket team.

We enjoy Right to freedom of Speech, Expression, Movement, belief and so on.

All of the activities, if forced, in some way or the other violate these fundamental rights.

A law that makes these compulsory can be challenged in Supreme Court or High courts.

34. Solution: d)

The Preamble in its present form reads:

“We, THE PEOPLE OF INDIA, having solemnly resolved to constitute India into a SOVEREIGN
SOCIALIST SECULAR DEMOCRATIC REPUBLIC and to secure to all its citizens.......”

This clearly tells that the source of authority of the constitution is the people of India.

A democratic polity, as stipulated in the Preamble, is based on the doctrine of popular sovereignty,
that is, possession of supreme power by the people.

http://www.insightsonindia.com                                                                              INSIGHTS   Page  11  


 
INSIGHTS  MOCK  TEST  SERIES  2015:  TEST  –  30  SOLUTIONS  
 
Universal adult franchise, periodic elections, rule of law, independence of judiciary, and absence of
discrimination on certain grounds are the manifestations of the democratic character of the Indian
polity.

35. Solution: c)

Public-Interest Litigation is litigation for the protection of the public interest. In Indian law,
Article 32 of the Indian constitution contains a tool which directly joins the public with judiciary.

A PIL may be introduced in a court of law by the court itself (suo motu), rather than the aggrieved
party or another third party. For the exercise of the court's jurisdiction, it is not necessary for the
victim of the violation of his or her rights to personally approach the court. In a PIL, the right to
file suit is given to a member of the public by the courts through judicial activism. The member of
the public may be a non-governmental organization (NGO), an institution or an individual.

The Supreme Court of India, rejecting the criticism of judicial activism, has stated that the
judiciary has stepped in to give direction because due to executive inaction, the laws enacted by
Parliament and the state legislatures for the poor since independence have not been properly
implemented.

Subodh Markandeya well known Senior Advocate of Supreme court of India and Judicial activist
believes that public interest litigation is the principal legal remedy For a common man and it is
main weapon of judicial activist .

36. Solution: c)

Any provision that changes the composition of the legislative bodies will require a constitutional
amendment. It is a major change and hence will require a higher majority in Parliament.

A political party is a self-governing voluntary body and is not a legislative or executive body in that
sense. A change in Societies Act will do for the political parties.

Hence, this is another solution advocated by parties to break the deadlock in Parliament over
women reservation.

http://www.insightsonindia.com                                                                              INSIGHTS   Page  12  


 
INSIGHTS  MOCK  TEST  SERIES  2015:  TEST  –  30  SOLUTIONS  
 
37. Solution: c)

Third Schedule Forms of Oaths or Affirmations for MPs, Ministers, Constitutional functionaries
etc.

Fourth Schedule - Allocation of seats in the Rajya Sabha to the states and the union territories.

Fifth Schedule - Provisions relating to the administration and control of scheduled areas and
scheduled tribes.

Sixth Schedule - Provisions relating to the administration of tribal areas in the states of Assam,
Meghalaya, Tripura and Mizoram.

Seventh Schedule - Division of powers between the Union and the States in terms of List I
(Union List), List II (State List) and List III (Concurrent List). Presently, the Union List contains
100 subjects (originally 97), the state list contains 61 subjects (originally 66) and the concurrent
list contains 52 subjects (originally 47).

Eighth Schedule - Languages recognized by the Constitution.

38. Solution: a)

None of the options is direct. You can work out by elimination.

Option B is false because the representatives are not supposed to command administration.

Option C is wrong because no branch of the government is subordinate to any other branch.

Option D is wrong because administration is directly controlled by Ministers as head of Ministries


and departments.

Administration is a part of the executive branch. Hence, Option A.

39. Solution: d)

http://www.insightsonindia.com                                                                              INSIGHTS   Page  13  


 
INSIGHTS  MOCK  TEST  SERIES  2015:  TEST  –  30  SOLUTIONS  
 
Unlike the Rajya Sabha, the Lok Sabha is not a continuing chamber. Its normal term is five years
from the date of its first meeting after the general elections, after which it automatically dissolves.

However, the President is authorised to dissolve the Lok Sabha at any time even before the
completion of five years and this cannot be challenged in a court of law.

Further, the term of the Lok Sabha can be extended during the period of national emergency be a
law of Parliament for one year at a time for any length of time. However, this extension cannot
continue beyond a period of six months after the emergency has ceased to operate.

40. Solution: b)

The Table of Precedence is related to the rank and order of the officials of the Union and State
Governments. The present notification on this subject was issued on 26 July, 1979. This
notification superseded all the previous notifications and was also amended many times. The
updated version of the Table, containing all the amendments made therein so far (2013), is given
below:

1. President

2. Vice-President

3. Prime Minister

4. Governors of states within their respective states

5. Former presidents

5A. Deputy Prime Minister

6. Chief Justice of India

Speaker of Lok Sabha

7. Cabinet Ministers of the Union

Chief Ministers of States within their respective States

Deputy Chairman, Planning Commission

Former Prime Ministers

Leaders of Opposition in Rajya Sabha and Lok Sabha

41. Solution: b)

http://www.insightsonindia.com                                                                              INSIGHTS   Page  14  


 
INSIGHTS  MOCK  TEST  SERIES  2015:  TEST  –  30  SOLUTIONS  
 
Article 355 imposes two duties on the Centre: (a) to protect every state against external aggression
and internal disturbance; and (b) to ensure that the government of every state is carried on in
accordance with the provisions of the Constitution.

This shows that the Union government has been given prominence over the state government. This
shows the federal character with a Union bias i.e. quasi-federal nature of the Indian polity. The
Union government has a responsibility to ensure the territorial integrity and security of the nation.

42. Solution: d)

This is a very comprehensive article that discussed every aspect of Lok Adalats. Go through the
article selectively.

http://www.legalserviceindia.com/articles/lok_a.htm

43. Solution: d)

Service tax is a tax levied by the government on service providers on certain service transactions,
but is actually borne by the customers. It is categorized under Indirect Tax and came into existence
under the Finance Act, 1994.

It is charged to the individual service providers on cash basis, and to companies on accrual basis.
This tax is payable only when the value of services provided in a financial year is more than Rs 10
lakh. This tax is not applicable in the state of Jammu & Kashmir.

44. Solution: a)

The 2nd schedule of the constitution fixes the allowances, privileges, emoluments of President,
Governor, CJI and other functionaries. It can be revised only after amending this part of the
constitution. These are not subject to the annual vote of the Parliament.

91st amendment fixed the maximum strength of the Council of Ministers at 15% of the strength of
the LS.

While the manner of election of the Panchs is fixed by constitution in form of adult franchise, the
state legislature decides on the manner of election of the Sarpanch.

45. Solution: d)

Annual budget is a money bill.

A Money Bill can be introduced only in the Lok Sabha and not in the Rajya Sabha.

Rajya Sabha cannot amend or reject a Money Bill. It should return the bill to the Lok Sabha within
14 days, either with recommendations or without recommendations.

The Lok Sabha can either accept or reject all or any of the recommendations of the Rajya Sabha. In
both the cases, the money bill is deemed to have been passed by the two Houses.

http://www.insightsonindia.com                                                                              INSIGHTS   Page  15  


 
INSIGHTS  MOCK  TEST  SERIES  2015:  TEST  –  30  SOLUTIONS  
 

46. Solution: b)

US is hoping to strengthen democracy in Tunisia especially after Beji Caid Essebsi became the first
democratically elected President in 60-year history of Tunisia.

By granting the MNNA status, both countries will enhance military cooperation between
them. The strong relations between both democracies will help to maintain peace and security in
Tunisia as it was the birthplace of Arab Spring (Jasmine revolution) revolts across the Middle East
in 2011.

It will also help Tunisia to counter the rising threat of jihadist groups especially ISIS in the region.

47. Solution: a)

Indian-origin Sunita Williams is among four veteran astronauts chosen by National Aeronautics
and Space Administration (NASA) to fly the first commercial space vehicles.

NASA had chosen Williams for the astronaut programme in 1998. Prior to this she was a US Navy
captain and was a helicopter pilot with more than 3,000 flight hours experience in more than 30
different aircraft. Presently, she ranks 6th on the all-time US endurance list and 2nd all-time for a
female astronaut. She holds the world records for most spacewalk time by a woman astronaut (50
hours, 40 minutes) and most spacewalks by a woman astronaut (seven times).

Kalpana Chawla was the first Indian-American astronaut and first Asian woman in space. She first
flew on Space Shuttle Columbia in 1997 as a mission specialist and primary robotic arm operator.

48. Solution: a)

As Iran has observer status in the SCO, it will serve as a platform for India to boost trade through
the Iranian ports of Bandar Abbas and Chabahar. These ports are considered as India’s gateway to
Central Asia through International North-South Transport Corridor (INSTC).

Thus, it will open up trade, energy sector and strategic transit routes for India between Russia,
Central Asia and China.

SCO will provide platform for the security grouping for India and Pakistan especially on issue to
counter terrorism.

It will also provide a valuable interface to engage with security concern of Afghanistan especially
its members including India due to pullout international troops.

http://www.insightsonindia.com                                                                              INSIGHTS   Page  16  


 
INSIGHTS  MOCK  TEST  SERIES  2015:  TEST  –  30  SOLUTIONS  
 
SCO may also serve as guarantor for projects such as the Turkmenistan-Afghanistan-Pakistan-
India (TAPI) and Iran-Pakistan-India (IPI) pipelines, which are held by India due to security
concerns.

Balance in International Politics: In this politically polarised world, SCO will play an important
role in counter-balancing India’s perceived tilt on security issues towards US and its allies. Thus, it
can help to maintain full balance of India’s relations with the great powers globally.

49. Solution: c)

RATS SCO, headquartered in Tashkent (Uzbekistan), is a permanent organ of the Shanghai


Cooperation Organization which serves to promote cooperation of Member States against
terrorism, separatism, and extremism, which it refers to as "the three evil forces."

RATS SCO includes 8 Member States (the Republic of Kazakhstan, the People's Republic of China,
the Kyrgyz Republic, the Russian Federation, the Republic of Tajikistan, and the Republic of
Uzbekistan, India and Pakistan), three observers, and three dialogue partners.

To ensure the safety of border areas, the border defence cooperation mechanism under the
framework of RATS SCO was established. RATS SCO has also collected and distributed to its
Member States intelligence information regarding the use of the Internet by terrorist groups active
in the region to promote their ideas.

Understand the operations of SCO here http://www.un.org/en/sc/ctc/news/2014-10-


24_cted_shanghaicoop.html

http://ecrats.org/en/

50. Solution: a)

For Rice, alluvial rich soil is needed. With that warm and moist climate is also required.

For Millets, dry and less fertile soil also will do. That is why it is grown in semi-arid areas.

For Barley too warm and moist climate is needed. It can do without rich soil.

Tea requires cool climate even with a poor soil.

51. Solution: d)

This is the description of the Sarnath Pillar.

http://www.insightsonindia.com                                                                              INSIGHTS   Page  17  


 
INSIGHTS  MOCK  TEST  SERIES  2015:  TEST  –  30  SOLUTIONS  
 
https://www.khanacademy.org/humanities/art-asia/south-asia/buddhist-art2/a/lion-capital-
ashokan-pillar-at-sarnath

The pillars have four component parts in two pieces: the three sections of the capitals are made in
a single piece, often of a different stone to that of the monolithic shaft to which they are attached
by a large metal dowel.

52. Solution: b)

Antarctica has the highest mean elevation among all continents.

All open oceans have same mean level because they are all connected with each other.

The deepest part of the ocean is more than 12,000 Km deep (Mariana trench); whereas Mount
Everest is about 8.3 km high.

53. Solution: a)

Turkey and Lebanon are comparatively older democracies than Egypt, Libya or Tunisia which
became democracies right after the Arab Spring.

In 2011, a revolution resulted in the overthrow of the autocratic President Zine El Abidine Ben Ali
followed by Tunisia’s first free elections. Since then, Tunisia has been consolidating democracy.

On 7 July 2012, Libyans voted in parliamentary elections, the first free elections in almost 40
years.

The Turkish War of Independence (1919–22), initiated by Mustafa Kemal Atatürk and his
colleagues in Anatolia, resulted in the establishment of the modern Republic of Turkey in 1923,
with Atatürk as its first president.

54. Solution: a)

https://www.wto.org/english/thewto_e/whatis_e/tif_e/org5_e.htm

The WTO’s main functions are to do with trade negotiations and the enforcement of negotiated
multilateral trade rules

Much attention is paid to the special needs and problems of developing and transition economies.
The WTO Secretariat’s Training and Technical Cooperation Institute organizes a number of
programmes to explain how the system works and to help train government officials and
negotiators.

A declaration also recognizes the contribution that trade liberalization makes to the growth and
development of national economies.

55. Solution: b)

http://www.insightsonindia.com                                                                              INSIGHTS   Page  18  


 
INSIGHTS  MOCK  TEST  SERIES  2015:  TEST  –  30  SOLUTIONS  
 
The Sushruta Samhita is commonly dated to the 6th century B.C., and is attributed to the
physician Sushruta (meaning ‘very famous’ in Sanskrit). The Sushruta Samhita’s most well-known
contribution to plastic surgery is the reconstruction of the nose, known also as rhinoplasty.

It may also be pointed out that the Sushruta Samhita is also one of the foundational texts of the
Ayurveda, the traditional medical system of India. Therefore, the Sushruta Samhita contains more
than just the description of plastic surgery procedures.

http://www.theguardian.com/world/2014/oct/28/indian-prime-minister-genetic-science-existed-
ancient-times

56. Solution: a)

The subtropical humid climate is characterized by hot summers and mild winters, though some of
the regions in this climatic type have rather cold winters. This type of climate is, however, marked
by variations within it.

For example, precipitation regime (seasonal distribution) is not the same everywhere. In some
areas, the summer has the maximum precipitation, while in others there is a distinct winter
precipitation maximum due to cyclonic storms and frontal activity. However, in humid subtropical
climate more or less rain falls throughout the year.

You can read more details at http://www.preservearticles.com/2011111717355/useful-notes-on-


subtropical-humid-climate-cf-or-china-type-of-climate.html

57. Solution: d)

Coke is a fuel with few impurities and high carbon content, usually made from coal. It is the solid
carbonaceous material derived from destructive distillation of low-ash, low-sulfur bituminous coal.
Cokes made from coal are grey, hard, and porous. While coke can be formed naturally, the
commonly used form is man-made. The form known as petroleum coke, or pet coke, is derived
from oil refinery coker units or other cracking processes.

Coke is used in preparation of producer gas which is a mixture of carbon monoxide and nitrogen.
Producer gas is produced by passing air over red hot coke. Coke is also used to manufacture water
gas.

58. Solution: a)

The Roaring Forties are strong westerly winds found in the Southern Hemisphere, generally
between the latitudes of 40 and 50 degrees. The strong west-to-east air currents are caused by the
combination of air being displaced from the Equator towards the South Pole and the Earth's
rotation, and there are few landmasses to serve as windbreaks.

The Roaring Forties were a major aid to ships sailing from Europe to the East Indies or Australia
during the Age of Sail, and in modern usage are favoured by yachtsmen on round-the-world
voyages and competitions. The boundaries of the Roaring Forties are not consistent, and shift

http://www.insightsonindia.com                                                                              INSIGHTS   Page  19  


 
INSIGHTS  MOCK  TEST  SERIES  2015:  TEST  –  30  SOLUTIONS  
 
north or south depending on the season. Similar but stronger conditions occur in more southerly
latitudes and are referred to as the Furious Fifties and Shrieking or Screaming Sixties.

59. Solution: d)

Bengal revolutionaries during the freedom struggle took shelter in Tripura.

Moreover, communism in the state had its beginnings in the pre-independence era, inspired by
freedom struggle activities in Bengal, and culminating in regional parties with communist
leanings.

It capitalised on the tribal dissatisfaction with the mainstream rulers, and has been noted for
connection with the "sub-national or ethnic searches for identity

60. Solution: c)

http://www.nytimes.com/2010/04/07/science/07methane.html?_r=0

Methane has no smell. It is the main ingredient of natural gas, but the smell that consumers
associate with natural gas is actually a chemical added by the gas company to make leaks obvious.
Humans cannot sense methane itself, although at times it is present with another gas, hydrogen
sulfide, that has an evident stink. But because the methane is often pure, miners long ago began
carrying canaries to work with them, knowing that when the birds showed signs of distress it was
time to get out.

The main method for getting methane out of mines is to pump fresh air in, so much that visitors
can often feel a breeze.

61. Solution: a)

Hard water contains considerable amount of dissolved carbonates of magnesium & Calcium in
room temperature.

When they are boiled inside boilers the decomposed calcium & magnesium carbonates (solids)
blocks the pipe lines inside them which leads the pressure inside the boiler to increase rapidly
without control. This may cause the boilers to rupture.

This blocking inside the pipes does not suddenly happen in a day or two. It happens very slowly. It
is expensive task to clean boilers regularly. Therefore, hard water is not used inside boilers.

62. Solution: b)

A keystone species is a species that has a disproportionately large effect on its environment relative
to its abundance.

http://www.insightsonindia.com                                                                              INSIGHTS   Page  20  


 
INSIGHTS  MOCK  TEST  SERIES  2015:  TEST  –  30  SOLUTIONS  
 
Such species are described as playing a critical role in maintaining the structure of an ecological
community, affecting many other organisms in an ecosystem and helping to determine the types
and numbers of various other species in the community.

The role that a keystone species plays in its ecosystem is analogous to the role of a keystone in an
arch. While the keystone is under the least pressure of any of the stones in an arch, the arch still
collapses without it.

63. Solution: a)

An anti-cyclone -- also known as a high pressure area -- is a large atmospheric circulation system
with the wind flowing clockwise around it in the Northern Hemisphere, and counter-clockwise in
the Southern Hemisphere.

Anticyclones form from air masses cooling more than their surroundings, which causes the air to
contract slightly making the air denser. Since dense air weighs more, the weight of the atmosphere
overlying a location increases, causing increased surface air pressure.

The air mass cooling that results in an anticyclone forming can be caused by either conduction as
the air flows over a relatively cool ocean surface, or through the loss of infrared radiation over land
during the fall, winter, or spring when little sunlight is available to warm the air mass.

The strongest anticyclones occur over snow-covered portions of Asia and North America in the
winter when clear, dry air masses cool from a loss of infrared radiation, while little sunlight is
absorbed to offset that infrared cooling.

64. Solution: c)

The principal components of volcanic gases are water vapor (H2O), carbon dioxide (CO2), sulfur
either as sulfur dioxide (SO2) (high-temperature volcanic gases) or hydrogen sulfide (H2S) (low-
temperature volcanic gases), nitrogen, argon, helium, neon, methane, carbon monoxide and
hydrogen.

Understanding gases dissolved in magma is critical in understanding why volcanoes erupt. Bodies
of magma rise in the crust until they reach a point of neutral buoyancy. The expansion of gases
brings the magma closer to the surface and drives eruptions. The interaction between the viscosity

http://www.insightsonindia.com                                                                              INSIGHTS   Page  21  


 
INSIGHTS  MOCK  TEST  SERIES  2015:  TEST  –  30  SOLUTIONS  
 
and temperature of the magma and the gas content determines if an eruption will be effusive or
explosive.

65. Solution: a)

The streams within a drainage basin form certain patterns, depending on the slope of land,
underlying rock structure as well as the climatic conditions of the area.

These are dendritic, trellis, rectangular, and radial patterns. The dendritic pattern develops where
the river channel follows the slope of the terrain. The stream with its tributaries resembles the
branches of a tree, thus the name dendritic. A river joined by its tributaries, at approximately right
angles, develops a trellis pattern.

A trellis drainage pattern develops where hard and soft rocks exist parallel to each other. A
rectangular drainage pattern develops on a strongly jointed rocky terrain. The radial pattern
develops when streams flow in different directions from a central peak or dome like structure.

A combination of several patterns may be found in the same drainage basin.

66. Solution: c)

The Brahmaputra rises in Tibet east of Mansarowar lake very close to the sources of the Indus and
the Satluj. It is slightly longer than the Indus, and most of its course lies outside India. It flows
eastwards parallel to the Himalayas. On reaching the Namcha Barwa (7757 m), it takes a ‘U’ turn
and enters India in Arunachal Pradesh through a gorge.

Here, it is called the Dihang and it is joined by the Dibang, the Lohit, the Kenula and many other
tributaries to form the Brahmaputra in Assam. In Tibet the river carries a smaller volume of water
and less silt as it is a cold and a dry area. In India it passes through a region of high rainfall.

Here the river carries a large volume of water and considerable amount of silt. The Brahmaputra
has a braided channel in its entire length in Assam and forms many riverine islands.

67. Solution: d)
http://www.insightsonindia.com                                                                              INSIGHTS   Page  22  
 
INSIGHTS  MOCK  TEST  SERIES  2015:  TEST  –  30  SOLUTIONS  
 
Western Ghats runs from north to south close to the western coast.

Most of the major rivers of the Peninsula such as the Mahanadi, the Godavari, the Krishna and the
Kaveri flow eastwards and drain into the Bay of Bengal. These rivers make deltas at their mouths.
There are numerous small streams flowing west of the Western Ghats.

The Narmada and the Tapi are the only long rivers, which flow west and make estuaries. The
drainage basins of the peninsular rivers are comparatively small in size.

68. Solution: d)

Due to the curvature of the earth, the amount of solar energy received varies according to
latitude. As a result, air temperature decreases from the equator towards the poles. As one goes
from the surface of the earth to higher altitudes, the atmosphere becomes less dense and
temperature decreases. The hills are therefore cooler during summers.

The pressure and wind system of any area depend on the latitude and altitude of the place.
Thus it influences the temperature and rainfall pattern. The sea exerts a moderating influence on
climate: As the distance from the sea increases, its moderating influence decreases and the
people experience extreme weather conditions. This condition is known as continentality (i.e. very
hot during summers and very cold during winters).

Ocean currents along with onshore winds affect the climate of the coastal areas, For example,
any coastal area with warm or cold currents flowing past it, will be warmed or cooled if the winds
are onshore.

69. Solution: d)

These low-pressure systems, originate over the Mediterranean Sea and western Asia and move into
India, along with the westerly flow. They cause the much-needed winter rains over the plains and
snowfall in the mountains. Although the total amount of winter rainfall locally known as
‘mahawat’ is small, they are of immense importance for the cultivation of ‘rabi’ crops.

The peninsular region does not have a well-defined cold season. There is hardly any noticeable
seasonal change in temperature pattern during winters due to the moderating influence of the sea.

70. Solution: d)

The reason is orographic rainfall on the western side of the Western Ghats. Clouds condense and
precipitate because of their collision with the western slopes of the mountains.

Moreover, the western slopes are highly continuous. So, few clouds can escape to the other side of
the mountain.

Hence, lesser rainfall and lesser vegetation.

71. Solution: d)

India is known for its herbs and spices from ancient times. Some 2,000 plants have been described
in Ayurveda and at least 500 are in regular use. The World Conservation Union’s Red list has

http://www.insightsonindia.com                                                                              INSIGHTS   Page  23  


 
INSIGHTS  MOCK  TEST  SERIES  2015:  TEST  –  30  SOLUTIONS  
 
named 352 medicinal plants of which 52 are critically threatened and 49 endangered. The
commonly used plants in India are:

Sarpagandha : Used to treat blood pressure; it is found only in India.

Jamun : The juice from ripe fruit is used to prepare vinegar which is carminative and diuretic, and
has digestive properties. The powder of the seed is used for controlling diabetes.

Arjun : The fresh juice of leaves is a cure for earache. It is also used to regulate blood pressure.

Babool : Leaves are used as a cure for eye sores. Its gum is used as a tonic.

Neem : Has high antibiotic and antibacterial properties.

Tulsi Plant : Is used to cure cough and cold.

Kachnar : Is used to cure asthma and ulcers. The buds and roots are good for digestive problems.

72. Solution: a)

Fault is a linear break in rocks of the earth’s crust along which there has been displacement in a
horizontal, vertical or oblique direction.

Fold is a bend in the rock strata resulting from compression of an area of the earth’s crust.

Geosyncline is a narrow, shallow, elongated basin with a sinking bottom in which a considerable
thickness of sediments was deposited by the rivers coming from Angara and Gondwanaland.

73. Solution: b)

Technically, all the resources within the territorial extent belong to the nation. The country has
legal powers to acquire even private property for public good.

There are international institutions which regulate some other resources outside territorial extent
of the nation. The oceanic resources beyond 200 km of the Exclusive Economic Zone belong to
open ocean and no individual country can utilise these without the concurrence of international
institutions.

74. Solution: b)

The Sundarban Delta derives its name from its name from the Sundari tree which grows well in
marshland.

Alternatively, it has been proposed that the name is a corruption of Samudraban, Shomudrobôn
("Sea Forest"), or Chandra-bandhe (name of a primitive tribe). However, the generally accepted
view is the one associated with Sundari trees.

It is the world’s largest and fastest growing delta. It is also the home of Royal Bengal tiger.

75. Solution: d)

http://www.insightsonindia.com                                                                              INSIGHTS   Page  24  


 
INSIGHTS  MOCK  TEST  SERIES  2015:  TEST  –  30  SOLUTIONS  
 
Materials in the environment which have the potential to satisfy human needs but human beings
do not have the appropriate technology to access these, are included among stock.

For example, water is a compound of two inflammable gases; hydrogen and oxygen, which can be
used as a rich source of energy.

But we do not have the required technical ‘know how’ to use them for this purpose. Hence, it can
be considered as stock.

76. Solution: a)

Soils in the drier areas are more alkaline and can be productive after proper treatment and
irrigation.

In some areas the salt content is very high and common salt is obtained by evaporating the water.
Due to the dry climate, high temperature, evaporation is faster and the soil lacks humus and
moisture.

Ploughing the soil in a wrong way i.e. up and down the slope form channels for the quick flow of
water that lead to soil erosion. So soil must be loosened and turned properly.

77. Solution: d)

Habitat destruction, hunting, poaching, over-exploitation, environmental pollution, poisoning and


forest fires are factors, which have led to the decline in India’s biodiversity.

Other important causes of environmental destruction are unequal access, inequitable consumption
of resources and differential sharing of responsibility for environmental well-being.

Over-population in third world countries is often cited as the cause of environmental degradation.
However, an average American consumes 40 times more resources than an average Somalian.

Similarly, the richest five per cent of Indian society probably cause more ecological damage
because of the amount they consume than the poorest 25 per cent. The former shares minimum
responsibilities for environmental well-being.

78. Solution: d)

Reserved forests are forest lands are protected from any further depletion.

Protected forests are regarded as most valuable as far as the conservation of forest and wildlife
resources.

Unclassed forests are other forests and wastelands belonging to both government and private
individuals and communities.

79. Solution: d)

http://www.insightsonindia.com                                                                              INSIGHTS   Page  25  


 
INSIGHTS  MOCK  TEST  SERIES  2015:  TEST  –  30  SOLUTIONS  
 
After Gandhiji’s martyrdom, Vinobha Bhave undertook padyatra to spread Gandhiji’s message
covered almost the entire country.

Once, when he was delivering a lecture at Pochampalli in Andhra Pradesh, some poor landless
villagers demanded some land for their economic well-being. Vinoba Bhave could not promise it to
them immediately but assured them to talk to the Government of India regarding provision of land
for them if they undertook cooperative farming.

Suddenly, Shri Ram Chandra Reddy stood up and offered 80 acres of land to be distributed among
80 land-less villagers. This act was known as ‘Bhoodan’. Later he travelled and introduced his
ideas widely all over India. Some zamindars, owners of many villages offered to distribute some
villages among the landless. It was known as Gramdan. However, many land-owners chose to
provide some part of their land to the poor farmers due to the fear of land ceiling act. This
Bhoodan-Gramdan movement initiated by Vinobha Bhave is also known as the Blood-less
Revolution.

80. Solution: d)

His principles of Dhamma were clearly stated in his Edicts. The main features of Asoka’s Dhamma
as mentioned in his various Edicts may be summed as follows:

• Service to father and mother, practice of ahimsa, love of truth, reverence to teachers and
good treatment of relatives.
• Prohibition of animal sacrifices and festive gatherings and avoiding expensive and
meaningless ceremonies and rituals.
• Efficient organization of administration in the direction of social welfare and maintenance
of constant contact with people through the system of Dhammayatras.
• Humane treatment of servants by masters and prisoners by government officials.
• Consideration and non-violence to animals and courtesy to relations and liberality to
Brahmins.
• Tolerance among all the religious sects.
• Conquest through Dhamma instead of through war.

81. Solution: d)

Mathura School of Art developed at Mathura in modern Uttar Pradesh is called the Mathura art. It
flourished in the first century A.D. In its early phase, the Mathura school of art developed on
indigenous lines.

The Buddha images exhibit the spiritual feeling in his face which was largely absent in the
Gandhara school. The Mathura school also carved out the images of Siva and Vishnu along with
their consorts Parvathi and Lakshmi.

The female figures of yakshinis and apsaras of the Mathura school were beautifully carved.

82. Solution: d)

http://www.insightsonindia.com                                                                              INSIGHTS   Page  26  


 
INSIGHTS  MOCK  TEST  SERIES  2015:  TEST  –  30  SOLUTIONS  
 
The king had also taken the advice of his minister, court-poet and the imperial court or avai.

The king was assisted by a large body of officials who were divided into five councils. They were
ministers (amaichar), priests (anthanar), military commanders (senapathi), envoys (thuthar) and
spies (orrar).

The military administration was also efficiently organized during the Sangam Age. Each ruler had
a regular army and their respective Kodimaram (tutelary tree). Land revenue was the chief source
of state’s income while custom duty was also imposed on foreign trade.

83. Solution: c)

Out of his nine years stay in India, he spent six years in the Gupta empire. He came to India by the
land route through Khotan, Kashgar, Gandhara and Punjab.

He visited Peshawar, Mathura, Kanauj, Sravasti, Kapilavastu, Kusinagara, Pataliputra, Kasi and
Bodh Gaya among other places. He returned by the sea route, visiting on the way Ceylon and Java.
The main purpose of his visit was to see the land of the Buddha and to collect Buddhist
manuscripts from India. He stayed in Pataliputra for three years studying Sanskrit and copying
Buddhist texts.

Fahien provides valuable information on the religious, social and economic condition of the Gupta
empire.

According to him, Buddhism was in a flourishing condition in the northwestern India but in the
Gangetic valley it was in a state of neglect. He refers to the Gangetic valley as the ‘land of
Brahmanism’

84. Solution: c)

During the British period cotton belts of India attracted the British and ultimately cotton was
exported to Britain as a raw material for their textile industries.

Champaran movement was started in 1917 in Bihar. This was started because farmers of that
region were forced to grow indigo on their land because it was necessary for the textile industries
which were located in Britain.

They were unable to grow foodgrains to sustain their families.

85. Solution: a)

The basic form of the Hindu temple comprises the following:

http://www.insightsonindia.com                                                                              INSIGHTS   Page  27  


 
INSIGHTS  MOCK  TEST  SERIES  2015:  TEST  –  30  SOLUTIONS  
 
(i) a cave-like sanctum (garbhagriha literally ‘womb-house’), which, in the early temples, was a
small cubicle with a single entrance and grew into a larger chamber in time. The garbhagriha is
made to house the main icon which is itself the focus of much ritual attention;

(ii) the entrance to the temple which may be a portico or colonnaded hall that incorporates space
for a large number of worshippers and is known as a mandapa;

(iii) from the fifth century CE onwards, freestanding temples tend to have a mountain-like spire,
which can take the shape of a curving shikhar in North India and a pyramidal tower, called a
vimana, in South India;

(iv) the vahan, i.e., the mount or vehicle of the temple’s main deity along with a standard pillar or
dhvaj is placed axially before the sanctum. Two broad orders of temples in the country are
known— Nagara in the north and Dravida in the south.

86. Solution: a)

Cyrus the Great was the greatest conqueror of the Achaemenian Empire. He was the first
conqueror who led an expedition and entered into India.

The Persian invasion provided an impetus to the growth of Indo-Iranian commerce. Also, it
prepared the ground for Alexander’s invasion. The use of the Kharoshti script, a form of Iranian
writing became popular in northwestern India and some of Asoka’s edicts were written in that
script.

We are able to see the influence of Persian art on the art of the Mauryas, particularly the
monolithic pillars of Asoka and the sculptures found on them. The very idea of issuing edicts by
Asoka and the wording used in the edicts are traced to Iranian influence. In short, the Iranian
connection with India proved more fruitful than the short-lived Indo-Macedonian contact.

87. Solution: d)

Samudragupta was the greatest of the rulers of the Gupta dynasty. The Allahabad Pillar inscription
provides a detailed account of his reign. It refers to three stages in his military campaign

• Against some rulers of North India


• His famous Dakshinapatha expedition against South Indian rulers
• A second campaign against some other rulers of North India.

He did not destroy and annex South Indian kingdoms. Instead, he defeated the rulers but gave
them back their kingdoms. He only insisted on them to acknowledge his suzerainty.

88. Solution: d)

According to the Permanent Land revenue settlement the Zamindars were recognised as the
permanent owners of the land. They were given instruction to pay 89% of the annual revenue to
the state and were permitted to enjoy 11% of the revenue as their share. They were left independent
in the internal affairs of their respective districts.

http://www.insightsonindia.com                                                                              INSIGHTS   Page  28  


 
INSIGHTS  MOCK  TEST  SERIES  2015:  TEST  –  30  SOLUTIONS  
 
The Zamindars were required to issue Patta and Quabuliyats to the cultivators mentioning the area
of their land, and the amount of revenue to be paid by them to the state. The historians have
expressed divergent opinions about its merits and demerits.

By making the Zamindars the owners of the land, the settlement created a class of loyal land lords
who formed a stable element in the state. The permanent settlement secured the political support
of the Zamindars of Bengal who stood loyal during the great mutiny of 1857.

89. Solution: d)

The Chalukya administration was highly centralized unlike that of the Pallavas and the Cholas.
Village autonomy was absent under the Chalukyas.

The Chalukyas had a great maritime power. Pulakesin II had 100 ships in his navy. They also had a
small standing army.

The village administration was carried on by the village headmen in the Rashtrakuta
administration. However, the village assemblies played a significant role in the village
administration.

90. Solution: b)

The English East India Company was established in 1600 and the Charter was issued by Queen
Elizabeth of England.

Captain Hawkins arrived at the royal court of Jahangir in 1609 to seek permission to establish
English trading centre at Surat. But it was refused by the Mughal Emperor due to Portuguese
pressure.

Later in 1612, Jahangir issued a farman (permission letter) to the English and they established a
trading factory at Surat in 1613.

91. Solution: a)

The Third Carnatic War ended with the French defeat. The French agreed to confine its activities
in Pondicherry, Karaikkal, Mahe and Yenam. Thus the Anglo-French rivalry came to a close with
British success and French failure.

The causes for the French failure can be summed up as follows:

• Britishers were superior in commerce and naval power.


• Lack of support from the French government.
• French had support only in the Deccan but the English had a strong base in Bengal.
• English had three important ports – Calcutta, Bombay and Madras but French had only
Pondicherry.
• Difference of opinion between the French Generals.
• England’s victory in the European wars decided the destiny of the French in India.

http://www.insightsonindia.com                                                                              INSIGHTS   Page  29  


 
INSIGHTS  MOCK  TEST  SERIES  2015:  TEST  –  30  SOLUTIONS  
 
92. Solution: d)

Mahatma Gandhi protested against the Communal Award and went on a fast unto death in the
Yeravada jail on 20 September 1932.

Finally, an agreement was reached between Dr Ambedkar and Gandhi. This agreement came to be
called as the Poona Pact.

The British Government also approved of it. Accordingly, 148 seats in different Provincial
Legislatures were reserved for the Depressed Classes in place of 71 as provided in the Communal
Award. The third Round Table Conference came to an end in 1932.

93. Solution: d)

The Woods dispatch - The establishment of departments of public instructions in five provinces
and introduction of the pattern of grants in aid to encourage private participation in the field of
education were recommended.

Besides, the dispatch also laid emphasis on the establishment of schools for technical education,
teacher and women education. Over and above all these, the dispatch recommended the
establishment of one University each in Calcutta, Bombay and Madras, on the model of the
London University.

Consequently, within the next few years, the Indian education became rapidly westernized.

94. Solution: a)

In 1878, the Vernacular Press Act was passed. This Act empowered a Magistrate to secure an
undertaking from the editor, publisher and printer of a vernacular newspaper that nothing would
be published against the English Government. The equipment of the press could be seized if the
offence was committed. This Act crushed the freedom of the Indian press. This created adverse
public opinion against the British Government.

In the same year, the Arms Act was passed. This Act prevented the Indians to keep arms without
appropriate license. Its violation would be a criminal offence. The Europeans and the Anglo-
Indians were exempted from the operation of these legislations.

95. Solution: d)

Ripon believed that self-government is the highest and noblest principles of politics. Therefore,
Ripon helped the growth of local bodies like the Municipal Committees in towns and the local
boards in taluks and villages. The powers of municipalities were increased. Their chairmen were to
be non-officials. They were entrusted the care of local amenities, sanitation, drainage and water-
supply and also primary education.

District and taluk boards were created. It was insisted that the majority of the members of these
boards should be elected non-officials. The local bodies were given executive powers with financial
resources of their own. It was perhaps the desire of Ripon that power in India should be gradually
transferred to the educated Indians.

http://www.insightsonindia.com                                                                              INSIGHTS   Page  30  


 
INSIGHTS  MOCK  TEST  SERIES  2015:  TEST  –  30  SOLUTIONS  
 
96. Solution: a)

Swami Vivekananda participated at the Parliament of Religions held in Chicago (USA) in


September 1893 and raised the prestige of India and Hinduism very high.

He said that all religions teach the same thing and gave the philosophy of ‘vasudhaiv
kutumbakam’. It meant that we all belong to the earth and that entire humanity is our family. He
then argued on the essential unity of all religions. He also mentioned that Hinduism captures the
essence of all major religions and is more all-encompassing than other religions.

He asked the people to improve the lives of the poor and depressed classes. He believed that
service to mankind is service to God. Vivekananda preached the message of strength and self-
reliance.

97. Solution: d)

• The Moderates were able to create a wide national awakening among the people.
• They popularized the ideas of democracy, civil liberties and representative institutions.
• They explained how the British were exploiting Indians. Particularly, Dadabhai Naoroji in
his famous book Poverty and UnBritish Rule in India wrote his Drain Theory. He showed
how India’s wealth was going away to England in the form of: (a) salaries,(b)savings, (c)
pensions, (d) payments to British troops in India and (e) profits of the British companies. In
fact, the British Government was forced to appoint the Welby Commission, with Dadabhai
as the first Indian as its member, to enquire into the matter.
• Some Moderates like Ranade and Gokhale favoured social reforms. They protested against
child marriage and widowhood.
• The Moderates had succeeded in getting the expansion of the legislative councils by the
Indian Councils Act of 1892.

98. Solution: d)

Two Home Rule Leagues were established, one by B.G. Tilak at Poona in April 1916 and the other
by Mrs. Annie Besant at Madras in September 1916. The aim of the Movement was to get self-
government for India within the British Empire. It believed freedom was the natural right of all
nations. Moreover, the leaders of the Home Movement thought that India’s resources were not
being used for her needs.

The two Leagues cooperated with each other as well with the Congress and the Muslim League in
putting their demand for home rule. While Tilak’s Movement concentrated on Maharashtra, Annie
Besant’s Movement covered the rest of the country. The Home Rule Movement had brought a new
life in the national movement. There was a revival of Swadeshi. Women joined in larger numbers.

99. Solution: a)

Warren Hastings realized the immediate need for introducing reforms by abolishing the dual
system.

http://www.insightsonindia.com                                                                              INSIGHTS   Page  31  


 
INSIGHTS  MOCK  TEST  SERIES  2015:  TEST  –  30  SOLUTIONS  
 
The East India Company decided to act as Diwan and to undertake the collection of revenue by its
own agents. Hence, the Dual System introduced by Robert Clive was abolished. As a measure After
the abolition of the Dual System, the responsibility of collecting the revenue fell on the shoulders
of the Company.

For that purpose, a Board of Revenue was established at Calcutta to supervise the collection of
revenue. English Collectors were appointed in each district.

100. Solution: d)

http://indiatoday.intoday.in/story/madrasas-derecognised-bjp-devendra-fadnavis-jnu-
rte/1/448785.html

The controversial move by the Maharashtra government to derecognise madrassas has created
furore.

The government argued that madrassas only gave religious instruction and did not impart formal
education in subjects such as science, maths, social science and English. Justifying it, state
Minority Affairs Minister Eknath Khadse said if madrassas started teaching these four subjects,
they would be considered as schools.

Khadse said, "Our Constitution says every child has the right to take formal education that
madrasas do not give. They are just educating children on religion and not giving them any formal
education."

http://www.insightsonindia.com                                                                              INSIGHTS   Page  32  


 
INSIGHTS  MOCK  TEST  -­‐  30            
 

INSIGHTS ON INDIA MOCK PRELIMINARY EXAM - 2015

INSIGHTS ON INDIA MOCK TEST - 30


GENERAL STUDIES
PAPER-I
Time Allowed: 2 Hours Maximum Marks: 200

INSTRUCTIONS
1. IMMEDITELY AFTER THE COMMENCEMENT OF THE EXAMINATION, YOU SHOULD
CHECK THAT THIS TEST BOOKLET DOES NOT HAVE ANY UNPRINTED OR TORN OR MISSING
PAGES OR ITEMS, ETC. IF SO, GET IT REPLACED BY A COMPLETE TEST BOOKLET.
2. You have to enter your Roll Number on the Test
Booklet in the Box provided alongside. DO NOT
Write anything else on the Test Booklet.
4. This Test Booklet contains 100 items (questions). Each item is printed only in English. Each item
comprises four responses (answers). You will select the response which you want to mark on the
Answer Sheet. In case you feel that there is more than one correct response, mark the response which
you consider the best. In any case, choose ONLY ONE response for each item.
5. You have to mark all your responses ONLY on the separate Answer Sheet provided. See directions in
the Answer Sheet.
6. All items carry equal marks.
7. Before you proceed to mark in the Answer Sheet the response to various items in the Test Booklet, you
have to fill in some particulars in the Answer Sheet as per instructions sent to you with your
Admission Certificate.
8. After you have completed filling in all your responses on the Answer Sheet and the examination has
concluded, you should hand over to the Invigilator only the Answer Sheet. You are permitted to take
away with you the Test Booklet.
9. Sheets for rough work are appended in the Test Booklet at the end.
10. Penalty for wrong answers :
THERE WILL BE PENALTY FOR WRONG ANSWERS MARKED BY A CANDIDATE IN THE
OBJECTIVE TYPE QUESTION PAPERS.
(i) There are four alternatives for the answer to every question. For each question for which a
wrong answer has been given by the candidate, one-third of the marks assigned to that
question will be deducted as penalty.
(ii) If a candidate gives more than one answer, it will be treated as a wrong answer even if one of
the given answers happens to be correct and there will be same penalty as above to that
question.
(iii) If a question is left blank, i.e., no answer is given by the candidate, there will be no penalty
for that question.

http://www.insightsonindia.com

INSIGHTS ON INDIA MOCK TEST SERIES FOR CIVIL SERVICES PRELIMINARY EXAM 2015

http://www.insightsonindia.com                                                          INSIGHTS   Page  1  


 
INSIGHTS  MOCK  TEST  -­‐  30            
 

This little flute of a reed thou hast carried over


hills and dales, and hast breathed through it
1. Jayaprakash Narayan’s Concept of Total
melodies eternally new........
Revolution was aimed at
a) Exposing the totalitarian nature of Thy infinite gifts come to me only on these
the incumbent government very small hands of mine. Ages pass, and still
b) Removing landlessness from rural thou pourest, and still there is room to fill.”
areas completely by Bhoodan and
forced acquisitions These are from the poems written in
c) Bringing a change in the existing
a. Savitri: A Legend and a Symbol - by
society in tune with the ideals of the
Sri Aurbindo
Sarvodaya
b. Mappings by Vikram Seth
d) Strengthening the local self-
c. Gitanjali - by Rabindranath Tagore
governing institutions on Gandhian
d. The Golden Threshold – by Sarojini
lines enabling them to counter
Naidu
increasing centralization in
governance

4. Consider the following about the


recently released Socio-economic Caste
2. In India the term ‘Buddhist circuit’ can
Census (SECC).
be used to refer to
1. Majority of households in India are
1. Racing track in Noida
deprived as per SECC.
2. Sarnath-Gaya-Varansi region
2. Majority of households are male-
3. Old Mahayana scriptures found in
headed.
Veshali
3. Over one-third of population living
Choose the correct answer using the codes in rural areas is illiterate.
below. 4. Landlessness is now rare in rural
India.
a) 1 and 2 only
b) 2 and 3 only
c) 1 and 3 only
d) All of the above Choose the correct answer using the codes
below.

a) 1 and 2 only
3. Consider the following lines. b) 2 and 3 only
c) 1 and 4 only
“Thou hast made me endless, such is thy
d) 3 and 4 only
pleasure. This frail vessel thou emptiest again
and again, and fillest it ever with fresh life.

http://www.insightsonindia.com                                                          INSIGHTS   Page  2  


 
INSIGHTS  MOCK  TEST  -­‐  30            
 
5. There has been a proposal to utilize the 7. Consider the following about the state of
Foreign Exchange reserves for financing malnourishment in India as per recently
infrastructure projects. Why is there released data by UNICEF.
concern about using Forex for such 1. Underweight Children
projects? 2. Underweight adolescent girls
1. Infrastructure projects are long-term 3. Stunted Children
and risky in nature.
2. Enough Forex reserves are needed to Majority of children (girls where applicable) in
avert a Balance of Payment crisis India fall under which of the above categories?
situation. a) 1 and 2 only
Which of the above is/are correct? b) 2 and 3 only
c) 1 and 3 only
a) 1 only d) 2 only
b) 2 only
c) Both 1 and 2 8. When we refer to the stabilizing market
d) None forces we generally refer to
a) Forces of Supply and demand
6. The Scheduled Tribes and Other b) Perfect competition in the market
Traditional Forest Dwellers c) Optimizing behaviour of individuals
(Recognition of Forest Rights) Act, d) All of the above
2006, also called as the Forest Rights
Act (FRA), 2006 confers which of the 9. Consider the following about
following rights on tribals? Rakhigarhi.
1. Rights over forest land patches 1. The site is the largest known city of
2. Rights over minor produce from the Indus civilization.
forests 2. It is older than Harappa and
3. Rights over timber produce from Mohenjodaro sites.
forest 3. It is situated on the dry bed of the
4. Rights over the wildlife in the forests Sarasvati river.
4. The site does not have urban
Choose the correct answer using the codes features like standards, coins,
below. drainage system.
a) 1 and 2 only Choose the correct answer using the codes
b) 2, 3 and 4 only below.
c) 1 and 4 only
d) All of the above a) 1 and 3 only
b) 2 and 4 only
c) 1, 2 and 3 only
d) 1, 3 and 4 only

http://www.insightsonindia.com                                                          INSIGHTS   Page  3  


 
INSIGHTS  MOCK  TEST  -­‐  30            
 
10. In the recent visit of Prime Minister to a. 1 only
the Central Asia republics, concerns b. 2 only
were raised on terrorist activities from c. Both 1 and 2
these republics as d. None
a) Many from the security agencies of
the Central Asian republics are
joining terrorist outfits 13. The recent Chinese stock market crash
b) There is an apprehension of State was due to
sponsored terrorism a. Poor state of economic reforms in
c) There is an apprehension of China
Military sponsored terrorism b. Slack regulation of stock markets by
d) The Central Asian republics are the the Chinese government
main base for all the terrorist c. The contagion of the Greece crisis
activities being run in West Asia d. None of the above

11. Which of these authorities is responsible


for GDP data calculation & which
14. Consider the following.
authority is responsible for revising GDP
1. Linfa
calculation methodology?
2. Chan-Hom
1. Central government
3. Nangka
2. Economic Adviser to Finance
Ministry How the above are related to China?
3. RBI
4. Central Statistical Office a. These are recently erupted volcanoes
in China.
Choose the correct answer using the codes b. These are typhoons that have
below. recently either hit China or are
nearing around it.
a. 3 and 1
c. These are parts of the remote
b. 3 and 2
Southern Province of China that
c. 4 and 1
borders Aksai China that were
d. 4 does both.
recently in news due to heightened
cross-border infiltrations.
12. The Indian constitution protects the
d. None of the above
rights of minorities because
1. In an electoral democracy, the will
of the majority prevails over the
minority.
2. All minority classes are socio-
economically vulnerable India.

Which of the above is/are true?

http://www.insightsonindia.com                                                          INSIGHTS   Page  4  


 
INSIGHTS  MOCK  TEST  -­‐  30            
 
15. Why is the Akash missile highly useful 17. India and United States have signed
for the Indian army that has recently inter-governmental agreement to
inducted it? implement the Foreign Account Tax
1. It is a surface-to-air missile, so it can Compliance Act (FATCA). It will help
hit targets from land. both countries to
2. It can cruise at supersonic speeds. 1. Promote transparency on tax matters
3. It is capable to neutralise aerial 2. Curb offshore tax evasions
targets like cruise missiles, fighter 3. Avoid problems related to double
jets and air-to- surface missiles. taxation of firms
4. It can engage multiple targets
simultaneously and can travel in Choose the correct answer using the codes
multiple directions. below.

Choose the correct answer using the codes a. 1 and 2 only


below. b. 2 and 3 only
c. 1 and 3 only
a. 1 and 2 only d. All of the above
b. 3 and 4 only
c. 1, 2 and 3 only 18. India should be economically interested
d. All of the above in Kazakhastan because
1. It is the World’s largest producer of
Uranium.
16. Uttarakhand has become the first Indian 2. It can benefit fertilizer sector in
state to install a system to detect India due to its abundance of
earthquakes and disseminate warnings. natural gas.
It is able to give advance warning for 3. Its hydrocarbon reserves can
earthquakes because supplement the oil economy of
1. The system is capable of detecting India.
harmless P waves that arrive earlier Choose the correct answer using the codes
than destructive S waves. below.
2. It has been deployed at the possible
focuses of past earthquake regions in a. 1 and 2 only
Uttarakhand. b. 2 and 3 only
c. 1 and 3 only
Which of the above is/are true? d. All of the above
a. 1 only
b. 2 only 19. Union Government has decided to
c. Both 1 and 2 develop all villages located within 10
d. None kilometres vicinity of the international
border on a priority basis with the help
of
1. Swachh Bharat Abhiyaan
http://www.insightsonindia.com                                                          INSIGHTS   Page  5  
 
INSIGHTS  MOCK  TEST  -­‐  30            
 
2. Promotion of sports activities in
border areas
3. Promotion of border tourism 22. The Reserve Bank of India exercises
4. Protection of heritage sites which of the following types of control
over banks?
Choose the correct answer using the codes 1. RBI sets the interest rates offered
below. by all domestic banks in India.
2. RBI can supersede strategic
a. 1 and 2 only decisions of the Board of Governors
b. 2 and 3 only of Public Sector Banks in
c. 1 and 4 only India.
d. All of the above 3. RBI regulates mergers and
acquisition of banks.

20. Japan’s Meiji-era sites historically Choose the correct answer using the codes
contributed in below.
a. Japan’s evolution from feudalism to a. 1 and 2 only
an industrialised modern economy b. 2 and 3 only
b. Development of fishing industry in c. 1 and 3 only
Japan d. All of the above
c. Tourism growth led by heritage
conservation in Japan
d. Sparking the technological 23. With regard to the Currency system in
innovation in Japan India, which of the following practices is
followed?
a. Level of currency volume is pegged at
21. In the Phillips curve what is the the amount of Gold reserves in the
relationship between rate of country
unemployment and inflation in the b. Level of currency circulation is
short-run? delinked from the level of metallic
a. Unemployment and inflation are not gold in the country
related to each other. c. Fresh currency cannot be printed if it
b. Decreasing Unemployment rate is not backed by a metallic form of
leads to lower inflation in the wealth
economy. d. Fresh currency cannot be printed if it
c. Decreasing Unemployment rate is not backed by foreign exchange
requires higher inflation in the reserves
economy.
d. Inflation can grow even at constant
levels of unemployment in the
economy.

http://www.insightsonindia.com                                                          INSIGHTS   Page  6  


 
INSIGHTS  MOCK  TEST  -­‐  30            
 
24. The calculation of national income 3. NRI deposits
covers which of the following?
1. Government consumption Choose the correct answer using the codes
expenditure below.
2. Net exports a. 1 and 2 only
3. Gross capital formation b. 2 and 3 only
Choose the correct answer using the codes c. 1 and 3 only
below. d. All of the above

a. 1 and 2 only
b. 2 and 3 only 27. What are ‘transfer payments’ given to
c. 1 and 3 only the public by the government?
d. All of the above a. Payment for services rendered in
present times
25. Why are subsidies given to general b. Payment that is not linked to the
public by the government inflationary in services in the present, and is a
nature? reward for services rendered in the
1. It increases the disposable income of past
people. c. Payment that is not linked to any
2. It does not tackle the supply side of service provided to government
goods. d. Both (b) and (c)
3. It inflates the fiscal deficit of the
government leading to greater
money supply.
28. The success of the Gold monetization
4. It distorts market mechanisms of
scheme will depend on
pricing.
1. The public perception of gold as an
Choose the correct answer using the codes asset
below. 2. Inflation in the economy
3. Ease of conversion of Gold into more
a. 1 and 4 only liquid assets
b. 2 and 3 only 4. Level of financial inclusion in the
c. 1, 3 and 4 only economy
d. All of the above
Choose the correct answer using the codes
below.
26. The difference between interest rates a. 1 and 2 only
between India and other countries can b. 2 and 4 only
affect which of the following in India? c. 1, 3 and 4 only
1. Currency exchange rate d. All of the above
2. Investment in the country

http://www.insightsonindia.com                                                          INSIGHTS   Page  7  


 
INSIGHTS  MOCK  TEST  -­‐  30            
 
29. How does the state government exercise
control over the local self-governing
bodies? 31. Consider the following.
1. By enacting laws, rules and 1. Government resigns if it fails to
regulations enact a public bill in Parliament.
2. By providing for provisions for 2. Prime Minister is a member of Lok
regular elections Sabha.
3. By releasing finances for 3. The council of Ministers reflects the
development projects socio-economic diversity of India.
4. By appointing the chairman and Which of these conventions has/have been
members of Panchayats and Ward codified into
councils rules/regulations/laws/provisions?
Choose the correct answer using the codes a. 1 only
below. b. 2 and 3 only
a. 1 and 2 only c. 1 and 3 only
b. 2 and 3 only d. None of the above
c. 1, 3 and 4 only
d. 1 and 3 only
32. Abolition of the legislative council in a
30. The constituent assembly of India state requires
1. Was elected based on universal adult a. the consent of the President and
franchise Governor concerned
2. Elected and appointed the Prime b. the consent of the Parliament and
Minister and other ministers in the the state legislature concerned
first government c. the consent of the State government
3. Enacted laws until the first official and State legislature
Parliament was elected d. the consent of Parliament, Governor
4. Was restricted in its powers by an and State legislature concerned
order of the British government
5. Supervised the functioning of the
Supreme Court and other 33. Which of the following can NOT be
subordinate courts forced on Indian citizens?
1. Singing national anthem
Choose the correct answer using the codes
2. Showing allegiance to all national
below.
sports teams
a. 1, 2 and 4 only 3. Healthy habits like regulated diets,
b. 2, 3 and 5 only exercise and Yoga
c. 1 and 3 only
Choose the correct answer using the codes
d. All of the above
below.

http://www.insightsonindia.com                                                          INSIGHTS   Page  8  


 
INSIGHTS  MOCK  TEST  -­‐  30            
 
a. 1 only
b. 2 and 3 only
c. 3 only 36. Providing for reservation of women in
d. None of the above which of the following bodies will NOT
require a constitutional amendment?
1. Lok Sabha
2. Rajya Sabha
34. Consider the following statements. 3. State Legislative Assembly
Assertion (A): The Constitution of India derives 4. State Legislative Council
its authority from the Parliament of India. 5. Political parties

Reason (R): It was enacted by the constituent Choose the correct answer using the codes
assembly of India which had legal and below.
democratic authority. a. All of the above
In the context of the statements above, which b. 3 and 4 only
of these is true? c. 5 only
d. None of the above
a) A and R both are true, and R is the
correct explanation for A. 37. Match these schedules of Constitution to
b) A and R both are true, and R is the what they contain.
NOT the correct explanation for A. 1. First Schedule – List of names of All
c) A is correct, R is incorrect. States and Union Territories
d) A is incorrect, R is correct. 2. Second Schedule – Powers of
President, Governor and Judges
3. Fourth Schedule – Allocation of seats
35. Consider the following about Public in Rajya Sabha
Interest litigation (PIL). 4. Seventh Schedule – Division of
1. PIL is a legal instrument. powers between Legislative,
2. It can be entertained by both Executive and Judiciary
administrative and judicial bodies.
Choose the correct answer using the codes
3. Representatives of victims can also
below.
file a PIL.
4. PIL can be filed only in social and a. 1, 2 and 4 only
environmental cases. b. 2 and 4 only
c. 1 and 3 only
Choose the correct answer using the codes
d. All of the above
below.

a. 1, 2 and 4 only
b. 2 and 3 only
c. 3 only
d. 3 and 4 only
http://www.insightsonindia.com                                                          INSIGHTS   Page  9  
 
INSIGHTS  MOCK  TEST  -­‐  30            
 
38. Parliament exercises control over the Choose the correct order using the codes below.
administration because
a. the executive is collectively a. 3124
responsible to the legislature b. 1324
b. it is the constitutional mandate of the c. 2134
representatives to command and d. 3241
control administrative affairs
c. executive branch of the government
is subordinate to the legislative 41. Article 355 of the Indian Constitution
branch of the government provides that ‘It shall be the duty of the
d. administration is not directly Union to protect every State against
controlled by the executive external aggression and internal
disturbance’. This shows the
a. Federal character of the Indian
39. Consider the following statements about constitution
the term of Lok Sabha. b. Quasi-federal Character of Indian
1. The President of India can reduce the polity
term of Lok Sabha. c. Democratic character of Indian
2. The speaker of Lok Sabha by himself polity
cannot shorten or extend the term of d. Unity and territorial integrity of
a Lok Sabha. India
3. Term of Lok Sabha can be extended
during emergency. 42. Lok Adalats are a useful to instrument to
reduce the burden on Judiciary in India.
Choose the correct answer using the codes Which of the following types of cases can
below. be handled by Lok Adalat?
1. Consumer grievance against food
a. 1 and 2 only standards in rural processed
b. 2 and 3 only products
c. 1 and 3 only 2. Cases involving property disputes
d. All of the above 3. Cases in involving Pension and other
transfer payment rights
4. Cases involving road accidents
40. Arrange the ranks of the following
functionaries from higher to lower in the Choose the correct answer using the codes
order of precedence. below.
1. Governor of a state within the state a. 1 and 2 only
2. Speaker of Lok Sabha b. 2 and 3 only
3. Former President c. 1 and 4 only
4. Chief Minister of a State outside a d. All of the above
state

http://www.insightsonindia.com                                                          INSIGHTS   Page  10  


 
INSIGHTS  MOCK  TEST  -­‐  30            
 
amendments, but the bill is not
passed in case of disagreement
43. Consider the following about Service b. it is sent to the President for re-
tax. consideration and then transferred to
1. It is an indirect tax. Lok Sabha for consideration
2. It is levied by the Central c. it is sent to a joint select committee
government. of the Parliament for re-examination
3. It is applicable to manufacturing as and re-introduction in Lok Sabha
well as services sector in India. d. None of the above
Choose the correct answer using the codes
below.
46. USA has recently granted Tunisia a
a. 1 and 2 only major Non-NATO Ally (MNNA) status.
b. 2 and 3 only This would allow both countries to
c. 1 and 3 only a. improve economic ties, reinforce
d. 2 only trade and reinvigorate mutual
investment
b. enhance military cooperation
44. Which of the following is fixed by the c. negotiate security contracts and
Constitution of India? exchange confidential information
1. Allowances of the President and d. improve mutual governance related
Governor matters
2. Maximum Size of Council of
Ministers 47. Consider the following.
3. Manner of election of the Sarpanch 1. She holds the record for the most
of the Gram Panchayat. spacewalks by a woman.
2. She holds the world record for the
Choose the correct answer using the codes
most spacewalk time by a woman
below.
astronaut.
a. 1 and 2 only 3. She is the first Indian woman
b. 2 and 3 only astronaut to serve NASA.
c. 1 and 3 only
Which of the above is/are true about Sunita
d. All of the above
Williams?

a. 1 and 2 only
45. If the Rajya Sabha does not agree to the b. 2 and 3 only
provisions of the Annual Budget then c. 1 and 3 only
a. it returns the bill to the Lok Sabha d. All of the above
with proposed amendments, which
then passes it or rejects the proposed

http://www.insightsonindia.com                                                          INSIGHTS   Page  11  


 
INSIGHTS  MOCK  TEST  -­‐  30            
 
48. Full Shanghai Corporation Organization 50. Which of the following crops do NOT
(SCO) membership to India will have need rich soil for its production?
significant benefits in 1. Barley
1. Combating terrorism 2. Millets
2. Opening up trade, energy sector and 3. Rice
strategic transit routes 4. Tea
3. Resolving strictly bilateral land
border disputes Choose the correct answer using the codes
below.
Choose the correct answer using the codes
below. a. 1, 2 and 4 only
b. 2 and 3 only
a. 1 and 2 only c. 2, 3 and 4 only
b. 2 and 3 only d. 1, 3 and 4 only
c. 1 and 3 only
d. All of the above
51. Pillars erected under the regime of
Ashok bear which of the following
49. Consider the following about the characteristics?
Regional Anti-terrorist Structure 1. They contain lessons in Dharma.
(RATS) of the Shanghai Corporation 2. They are monolithic.
Organization (SCO). 3. All pillars have been made by heavy
1. It is headquartered in Beijing, China. metals.
2. It is a permanent organ of the 4. They have been erected outside India
Shanghai Cooperation Organization. too.
3. It deals with extremism and
separatism too. Choose the correct answer using the codes
4. All member-states contribute to below.
intelligence gathering in RATS. a. 1, 3 and 4 only
Choose the correct answer using the codes b. 2 and 3 only
below. c. 2, 3 and 4 only
d. 1 and 2 only
a. 1 and 4 only
b. 2 and 3 only
c. 2, 3 and 4 only 52. Consider the following statements.
d. All of the above 1. Asia has the highest mean elevation
among all continents.
2. The distance of the deepest point in
the Ocean from mean sea level is
greater than that of the highest point
on land.

http://www.insightsonindia.com                                                          INSIGHTS   Page  12  


 
INSIGHTS  MOCK  TEST  -­‐  30            
 
3. All the open oceans of the world have b. 2 and 3 only
the same mean sea level. c. 1 and 3 only
d. 2 only
Choose the correct answer using the codes
below.

a. 1 and 2 only 55. In a conference recently, the PM talked


b. 2 and 3 only about technologies being highly
c. 1 and 3 only developed in ancient India. Which of
d. All of the above these technologies was the first to be
implemented in ancient India?
a. Technology of mass flight
53. After the Arab Spring, which of the b. Technology of plastic surgery
following West Asian or Middle-eastern c. Technology of surgery without the
countries turned democratic? use of hands and instruments
1. Tunisia d. Transplantation of organs between
2. Libya members of the same family
3. Egypt
4. Turkey
5. Lebanon 56. Consider the following about China type
of climate.
Choose the correct answer using the codes 1. High rainfall throughout the year
below. 2. Low temperature variation in
a. 1, 2 and 3 only summer and winter
b. 2 and 3 only 3. Distinct dry period
c. 4 and 5 only 4. Very cold winters
d. 2, 3, 4 and 5 only Choose the correct answer using the codes
below.

54. Consider the following objectives. a. 1 and 2 only


1. Protection of Intellectual property b. 2 and 3 only
rights. c. 1 and 4 only
2. Promotion of resource flow between d. 3 and 4 only
nations.
3. Ensuring technological
modernization of nations to 57. Consider the following about Coke.
specialize world trade. 1. It is used in coal as well as nuclear
Which of the above come under the mandate of power plants.
the World Trade Organization (WTO)? 2. It can be used to remove silica
content of the iron ore.
a. 1 and 2 only

http://www.insightsonindia.com                                                          INSIGHTS   Page  13  


 
INSIGHTS  MOCK  TEST  -­‐  30            
 
3. Coke contains higher percentage of a. 1 and 2 only
carbon by weight that coal. b. 2 and 3 only
c. 1 and 3 only
Choose the correct answer using the codes d. None of the above
below.

a. 1 and 2 only 60. Methane found in mines is dangerous


b. 2 and 3 only for the workers because
c. 1 and 3 only 1. When mixed with air it can become
d. 3 only explosive.
2. It can be poisonous even in open
mines.
3. Methane mixed with mine water can
58. Which of the following factors
become toxic for human
contribute to formation of Roaring
consumption.
Forties?
1. Air being displaced from Choose the correct answer using the codes
the Equator towards the South Pole below.
2. Earth's rotation
3. Easterlies a. 1 and 2 only
4. Ocean currents b. 2 and 3 only
c. 1 only
Choose the correct answer using the codes d. All of the above
below.

a. 1 and 2 only
b. 2 and 3 only 61. Boiler industries do not use hard water
c. 1 and 4 only for their operations. Why?
d. All of the above a. Hard water can result in scaling and
cracking of boilers.
b. Hard water blocks boiler operation.
59. Why was Tripura involved in the Indian c. Minerals in hard water react with
freedom struggle? boiler metal and replace it.
1. The Tripura tribals were forced to d. It erodes boiler metals at a very fast
pay high land rent by the British. pace.
2. Many tribals of Tripura were part of
the Indian British army that were
involved in 1857 mutiny.
3. The princely state of Tripura was
occupied by the British.

Choose the correct answer using the codes


below.

http://www.insightsonindia.com                                                          INSIGHTS   Page  14  


 
INSIGHTS  MOCK  TEST  -­‐  30            
 
62. Keystone species are important in an
ecosystem because
a. They determine the genetic diversity 65. A rectangular drainage pattern develops
of an ecological community. where
b. They are important in determining a. strongly jointed rock terrain is found
the ability of a large number of other b. soft and hard rocks exist parallel to
species to persist in the community. each other
c. They are found at the edge of an c. river channels follow the slope of the
ecosystem and signal the shift in terrain
ecosystems. d. streams flow in two parallel
d. They are rare in the ecosystem and directions from a central peak
stand at the top of the food web.

63. Anti-cyclonic conditions can be formed 66. The Brahmaputra River carries a large
when volume of water and silt in India, but
1. Atmospheric pressure is high. carries a smaller amount of both water
2. Temperature is low. and silt while coming from Tibet. Why?
3. Rainfall is high and persistent. 1. Tibet is a cold and dry area.
2. Brahmputra forms many riverine
Choose the correct answer using the codes islands in Tibet.
below. 3. In India it passes from the North-
a. 1 and 2 only eastern region which is a region of
b. 2 and 3 only high rainfall.
c. 1 only Choose the correct answer using the codes
d. All of the above below.

64. The principal gases dissolved in Magma a. 1 and 2 only


is/are b. 2 and 3 only
1. Water vapour c. 1 and 3 only
2. Methane d. All of the above
3. Carbon dioxide
4. Sulphur dioxide 67. Consider the following statements about
5. Ozone rivers in Peninsular India.
1. The main water divide in Peninsular
Choose the correct answer using the codes India is formed by the Western
below. Ghats.
a. 1 and 2 only 2. The drainage basin of Peninsular
b. 2, 3 and 4 only Rivers is smaller than Himalayan
c. 1 and 3 only Rivers.
d. 3, 4 and 5 only 3. Peninsular rivers also flow in rift
valleys and form estuaries.

http://www.insightsonindia.com                                                          INSIGHTS   Page  15  


 
INSIGHTS  MOCK  TEST  -­‐  30            
 
Choose the correct answer using the codes a. 1 and 2 only
below. b. 2 and 3 only
c. 1 and 3 only
a. 1 and 2 only d. All of the above
b. 2 and 3 only
c. 1 and 3 only
d. All of the above
70. Why are the western slopes of the
68. Pressure and wind system of an area Western Ghats covered with thick
depends on which of the following forests and not the eastern slopes?
factors? a. Eastern slopes are regions of high
1. Latitude pressure that repel clouds which
2. Altitude results in lesser rainfall
3. Distance from sea b. Soil in Eastern slopes is less fertile
4. Ocean currents than that of western slopes
5. Relief features c. Lesser number of rivers are found on
the eastern side which results in
Choose the correct answer using the codes lesser evaporation and precipitation
below. d. Eastern slope falls in the rain shadow
area
a. 1, 2 and 3 only
b. 2 and 3 only
c. 1 and 5 only
d. All of the above 71. Which of these Indian plants have anti-
microbial and anti-biotic properties?
1. Neem
2. Tulsi
69. A characteristic feature of the cold 3. Arjun
weather season over the northern plains 4. Jamun
is the inflow of cyclonic disturbances
from the west and the northwest. Choose the correct answer using the codes
Consider the following about it. below.
1. These are low-pressure systems
which originate over the a. 1 and 4 only
Mediterranean Sea and western Asia. b. 2 and 3 only
2. They cause winter rains over the c. 1 and 3 only
plains and snowfall in the d. 1 only
Mountains.
3. They help in the cultivation of ‘rabi’
crops.

Choose the correct answer using the codes


below.

http://www.insightsonindia.com                                                          INSIGHTS   Page  16  


 
INSIGHTS  MOCK  TEST  -­‐  30            
 
72. In Geographical terms a ‘Fault’ is 75. Which of the following are examples of
1. A linear break in rocks of the earth’s ‘stock’ resources?
crust in Horizontal or vertical 1. Extracting hydrogen from water to
direction produce energy
2. A break in rocks in Oblique direction 2. Extracting shale gas for diversifying
3. Bend in the rock strata energy basket
3. Employing natural gas to produce
Choose the correct answer using the codes electrical energy
below.
Choose the correct answer using the codes
a. 1 and 2 only below.
b. 2 and 3 only
c. 1 and 3 only a. 1 and 2 only
d. 1 only b. 2 and 3 only
c. 1 and 3 only
d. 1 only
73. Consider the following about the oceanic
resources that lie beyond the Exclusive
Economic Zone (EEZ) 76. As the District Agriculture Advisor you
1. It can be exploited only by the have visited a dry area and find the soil
countries that border the EEZ. to be sub-standard for production of
2. No individual country can utilize these crops. You will recommend which of the
resources without the concurrence of following to improve the productivity of
international institutions. the soil?
1. Make the soil more acidic
Which of the above is/are correct? 2. Use mulching for improving soil
a. 1 only nutrients
b. 2 only 3. Tighten the soil
c. Both 1 and 2 Choose the correct answer using the codes
d. None below.

a. 1 and 2 only
74. The Sundarban Delta derives its name b. 2 and 3 only
from c. 1 and 3 only
a. it being the world’s largest delta d. 2 only
b. a local sundari tree which grows in
marshy areas
c. its magnificent beauty that is localy
called sundar
d. it being a large forest nearing a gulf

http://www.insightsonindia.com                                                          INSIGHTS   Page  17  


 
INSIGHTS  MOCK  TEST  -­‐  30            
 
77. Developing countries face the problem Choose the correct answer using the codes
of environmental degradation even below.
more starkly than the developed
countries because of a. 1 and 2 only
1. Over-population b. 2 and 3 only
2. High incidence of Poverty c. 1 and 3 only
3. High consumption per capita d. 2 only
4. Capacity deficit in controlling factors
like forest fires, invasion of unwanted
ecological elements 80.Ashoka’s concept of Dhamma included
1. Prohibition of animal sacrifices
Choose the correct answer using the codes 2. Efficient organization of
below. administration for social welfare
a. 1 and 2 only 3. Avoiding expensive and meaningless
b. 2 and 3 only ceremonies
c. 4 only 4. Service to father and mother
d. 1, 2 and 4 only Choose the correct answer using the codes
below.

78. Forests and wastelands belonging to a. 1 and 2 only


both government and private b. 2, 3 and 4 only
individuals and communities are known c. 1 and 3 only
as d. All of the above
a. Reserved Forests
b. Protected Forests
c. National classed reserves 81. Which of the following distinguish
d. Unclassed forests Mathura school art from Gandhara art?
1. Mathura art developed completely on
indigenous lines unlike Gandhara
79. Under ‘Gramdan’ movement spread by art.
Vinoba Bhave 2. The Buddha images under Mathura
1. Any land greater than the legal art exhibited spiritual expressions
ceiling was forfeited by Zamindars to which was largely absent in the
the government. Gandhara school.
2. Villages were distributed to landless 3. Mathura school also carved the
poor people. images of other deities unlike
3. Villages were instructed to abandon Gandhara school.
living spaces demarcated on the basis Choose the correct answer using the codes
of caste. below.

http://www.insightsonindia.com                                                          INSIGHTS   Page  18  


 
INSIGHTS  MOCK  TEST  -­‐  30            
 
a. 1 and 2 only
b. 2 and 3 only 84. Champaran movement was started in
c. 1 and 3 only 1917 in Bihar due to
d. All of the above a. High land rent demanded by the
Britishers from the tenants
b. Lack of security of tenure for tenants
82. Consider the following statements about working in British plantations
administration in Sangam age. c. farmers of that region being forced to
1. Hereditary monarchy was the form grow indigo on their land
of government. d. farmers not being paid rent for using
2. There was no separate military wing their land for plantation crops
in the administration.
3. The state did not involve itself in
foreign trade. 85. In Hindu temples architecture, ‘Vahana’
4. The state did not rely on land is that part of the temple which is
revenue for income. a. the mount or vehicle of the temple’s
main deity
Choose the correct answer using the codes b. portico or colonnaded hall at the
below. entrance of the temple
a. 1 and 2 only c. a mountain-like spire at the front of
b. 2 and 4 only the temple
c. All of the above d. rhythmically projecting symmetrical
d. 1 only walls near the Garbhgriha of the
temple

86. Persian invasions by Cyrus and others


83. The famous Chinese pilgrim, Fahien
had which of the following implications
visited India during the reign of
for India?
Chandragupta II. The main purpose of
1. Boost in Indo-Iranian commerce
his visit was to
2. Kharoshti, an Iranian writing became
a. Understand the functioning of
popular in north-western India.
monarchical government in Gupta
3. It helped temporarily avert invasion
empire
by Alexander.
b. Experience the culture and heritage
of India and write a literary piece on Choose the correct answer using the codes
it below.
c. See the land of the Buddha and to
collect Buddhist manuscripts from a. 1 and 2 only
India b. 2 and 3 only
d. None of the above c. 1 and 3 only
d. All of the above

http://www.insightsonindia.com                                                          INSIGHTS   Page  19  


 
INSIGHTS  MOCK  TEST  -­‐  30            
 
87. Consider the following statements. Choose the correct answer using the codes
1. Samudragupta annexed all his below.
conquered territories of South India.
2. The North Indian conquests of a. 1 and 2 only
Samudragupta did not result in the b. 2 and 3 only
expansion of the Gupta Empire. c. 1 and 3 only
3. Feats of the Gupta Empire are only d. All of the above
recorded in the writings of foreign
visitors to India.
90. The English East India Company wanted
Choose the correct answer using the codes their first trading centre to be set-up at
below. Surat. The permission was however
a. 1 and 2 only refused by the Mughal emperor there
b. 2 and 3 only because
c. 1 and 3 only a. The Mughal Emperor was against the
d. None of the above trade practices of the British
b. There was portugese pressure on the
Emperor
c. He was not interested in foreign
88. Grant of pattas by zamindars to farmers trade and investment
was a main feature of the Permanent d. The British did not promise adequate
Settlement system. This grant was revenue from the trading centre to
widely not successful because the Emperor
a. British government later opposed the
move of granting pattas to farmers
b. A very large number of farmers did
not register themselves for the grant
c. There was a nexus between British 91. In the British-French rivalry to take
officials and Zamindars who allotted control of Indian trade resulted in
land amongst themselves French failure. Which of the following
d. The zamindars were not officially can be the possible causes of the failure
accountable to the British of the French?
government for grant of pattas 1. Commercial and naval superiority of
the British
2. England’s victory in the European
89. The Chalukyan administration was wars
characterized by 3. Lack of support of French from any
1. High centralization in administration Mughal emperor
2. Absence of Village autonomy
3. Maritime power and small standing Choose the correct answer using the codes
army below.

http://www.insightsonindia.com                                                          INSIGHTS   Page  20  


 
INSIGHTS  MOCK  TEST  -­‐  30            
 
a. 1 and 2 only 94. The Vernacular Press Act passed in 1878
b. 2 and 3 only ensured that
c. 1 and 3 only a. nothing would be published by the
d. All of the above English government in any
vernacular newspaper
b. nothing seditious could be published
92. The Poona pact of 1932 that affects the territorial integrity of
1. Was a formal outcome of the Third British India
Round Table Conference c. the vernacular newspapers were not
2. Ensured reservation for depressed exposed to pre-censorship by the
classes in all public institutions British government
3. Provided for the British government d. no Indian national freedom fighter
to take welfare measures for the could establish a vernacular press
depressed classes company in India

Choose the correct answer using the codes


below.
95. Consider the following statements.
a. 1 and 2 only
b. 2 and 3 only Assertion (A): Local self-government bodies
c. 1 only were not recognized officially till Indian
d. None of the above Independence.

Reason (R): The British did not enact any


special legislation for the governance of local
93. In 1854, Sir Charles Wood sent a bodies.
comprehensive dispatch as a grand plan
on education. The plan laid emphasis on In the context of the statements above, which
1. Education in vernacular languages of these is true?
2. Provision of education only by public a) A and R both are true, and R is the
institutions correct explanation for A.
3. Westernization of Indian education b) A and R both are true, and R is the
Choose the correct answer using the codes NOT the correct explanation for A.
below. c) A is correct, R is incorrect.
d) A is incorrect, R is correct.
a. 1 and 2 only
b. 2 and 3 only
c. 1 and 3 only
d. 3 only

http://www.insightsonindia.com                                                          INSIGHTS   Page  21  


 
INSIGHTS  MOCK  TEST  -­‐  30            
 
96. In the Parliament of Religions held in Choose the correct answer using the codes
Chicago (USA) in 1893, Swami below.
Vivekananda gave the message of
a. Essential unity of all religions a. 1 and 2 only
b. Superiority of Indian culture over all b. 2 and 3 only
other cultures in the world c. 1 and 3 only
c. A classless society guided by core d. 3 and 4 only
human values
d. All of the above
99. When Warren Hastings assumed the
administration of Bengal in 1772, he
97. Even though the moderates were not found it in utter chaos and thus
very popular among the extremists, they abolished the ‘dual system’. After the
succeeded on a number of fronts. They abolition of the dual system
were successful at a. The East India Company acted both
1. popularizing the ideas of democracy, as Diwan and collector of revenue by
civil liberties and representative its own agents
institutions b. The British handed over financial
2. explained British exploitation of the powers to Mughal administration
Indian economy and focussed only on revenue
3. getting the legislative councils collection
expanded for Indian benefit c. The British handed financial
administration to the Mughals and
Choose the correct answer using the codes fixed a share from revenue for every
below. year
d. None of the above
a. 1 and 2 only
b. 2 and 3 only
c. 1 and 3 only
d. All of the above 100. On what grounds do the National
and State government have control over
religious education imparted in minority
institutions?
98. The Home Rule Movement of 1916 did a) On grounds of protecting the
NOT emphasize culture of minorities
1. Revival of Swadeshi b) On grounds of promoting
2. Participation of Indians in self- scientific temper in society
governing institutions for India c) On grounds of national unity and
3. Complete erosion of British integrity
sovereignty over India d) The government cannot control
4. Separate electorates for all religious education imparted in
communities of India minority institutions.

http://www.insightsonindia.com                                                          INSIGHTS   Page  22  


 
INSIGHTS  MOCK  TEST  -­‐  30            
 

http://www.insightsonindia.com                                                          INSIGHTS   Page  23  


 

You might also like